Docsity
Docsity

Prepare for your exams
Prepare for your exams

Study with the several resources on Docsity


Earn points to download
Earn points to download

Earn points by helping other students or get them with a premium plan


Guidelines and tips
Guidelines and tips

Cardiology Clinical Medicine Assignment Questions., Exams of Nursing

Cardiology Clinical Medicine Assignment Questions.

Typology: Exams

2023/2024

Available from 06/19/2024

CarlyBlair
CarlyBlair 🇺🇸

4

(1)

1.2K documents

1 / 27

Toggle sidebar

Related documents


Partial preview of the text

Download Cardiology Clinical Medicine Assignment Questions. and more Exams Nursing in PDF only on Docsity! Cardiology Clinical Medicine Assignment Questions *Answer: A • According to Smartypance and Professor Miliron's pathophysiology lecture, Mitral stenosis is often associated with Rheumatic fever. Further, according to smartypance, it is a non-radiating (typically) diastolic murmur that is heard best at the apex of the heart with the bell of the stethoscope (low pitch) and it is characterized by an opening snap just after S2. Mitral stenosis is commonly associated with dyspnea upon exertion (as well as orthopnea, paroxysmal nocturnal dyspnea, etc.)* • Answer choice: B o Mitral Regurgitation is incorrect because it is asystolicmurmur (Professor Miliron's lecture) typically with a high pitch blowing holosystolic murmur at apex with split S2 that radiates to the axilla (Smartypance). • Answer choice: C o Ventricular Septal Defect is another systolic murmur best heard over Erb's point. According to smartypance the murmur is loud, holosystolic along the left sternal bo - A 20-year-old male presents to your primary care office for a work required physical. Reviewing past medical history, you learn he had rheumatic fever as a teenager. His only complaint today during the physical is occasional exertional dyspnea. During the auscultation of his heart you notice a non-radiating, diastolic, rumbling murmur with an opening snap after S2, heard best with the bell of your stethoscope at the apex of his heart. Based on this information what do you think is causing the murmur? A.) Mitral Stenosis B.) Mitral Regurgitation C.) Ventricular Septal Defect D.) Pulmonary stenosis E.) Not enough information A) [P-waves are usually hard to discern due to the rapid rate of >100bpm] B) [supraventricular, meaning the issue is occurring above the ventricles and reflects rapid activation of the ventricles which would show a narrow QRS complex] C) [while there is a narrow QRS complex in PSVTs, the rhythm is usually regular. This description would be more consistent with A-Fib] D) [this would be more indicative of a PVC, where PSVTs originate above the ventricles] E) [this would be more indicative of a 2nd degree heart block. PSVTs do not show an issue with blockage] - A 55 y/o woman comes to the ER with complaints of sudden lightheadedness, sweating, and heart palpitations. An EKG is performed and shows evidence of paroxysmal supraventricular tachycardia (PSVT). Which one of the following choices is a common key identifying feature seen on EKG to know this is a PSVT? A. P-waves are distinct and can be easily seen marching out on the EKG B. The QRS is narrow, meaning the arrhythmia originates within or above the Bundle of His C. The QRS complex would be narrow with an irregularly irregular rhythm D. A wife QRS complex occurring earlier than expected with an inverted T-wave following E. A sudden drop of QRS complex due to the ventricles not conducting all of the atrial impulses The correct answer is D. Aortic stenosis is most clearly heard at the second intercostal space at the right sternal border. This would be heart during systole because during this phase of the cardiac cycle blood is being sent through the aortic and pulmonic valves, if the aortic valve is stenosed then the restricted blood flow will create an aortic stenosis murmur. - A 47y/o M arrives to your office for evaluation of new onset dizziness. Patient states that he walks two miles every morning but over the last week he has noticed a feeling of dizziness when he completes the last portion of his walk which is a "pretty steep incline." Patient states the dizziness resolves once he stops walking but he wants to make sure it is nothing serious. You suspect that the patient may be experiencing aortic stenosis, if your suspicions are correct where on the chest would you hear this murmur and what phase of blood pressure would it occur in? A. 2nd intercostal space LSB; diastole B. 4th intercostal space RSB; diastole C. 3rd intercostal space RSB; systole D. 2nd intercostal space RSB; systole a. Bradycardia is LESS than 60 bpm, ST elevation MUST be "elevation at the J-point in 2 anatomically contiguous leads ≥1mm in all leads except V2-V3," this axis is normal b. This is NSR, but LVH is diagnosed popularly with: V1 S-wave PLUS V5-V6 R-wave ≥35mm, or R + S in any V lead >45mm, this axis is normal. *c. NSR is 60 - 100bpm, absent criteria for LVH/RVH, with axis between 0º - 90º* d. Bradycardia is LESS than 60 bpm, RVH is popularly diagnosed with: V1-V2 "tall R-waves"; V5-V6 "deep S-waves"; V1 R-wave >7mm; showing an R:S ratio >1 in V1 and V2 suggests RVH. RAD is when the axis is between 90º and 180º. e. Torsades de pointes is a polymorphic VT that is >100bpm and often results from QT-prolongation just before TdP and then death! - You just began your 15th hour of your ER rotation on the first day. A 19 y/o male comes in who just ate some random mushrooms off the sidewalk and is now scared his stupidity might kill him. You immediately obtain this EKG. What do you see? a. Bradycardia, mild ST elevation, normal axis b. NSR (normal sinus rhythm), LVH, normal axis c. NSR, normal LV, normal axis d. Bradycardia, RVH, RAD e. Torsades de Pointes Answer: B Why all the others are wrong Inferior wall a. ECG: Q waves & ST elevation in leads II, III, and AVF b. Coronary artery: RCA, left circumflex Lateral wall a. ECG: ST elevation in the lateral leads (I, aVL, V5-6) b. Coronary Artery: Left circumflex Posterior wall a. ECG: ST depression in V1 to V3 b. Coronary artery: RCA, left circumflex - Patient presents to the emergency room with crushing chest pain. Patient has Q wave abnormalities and ST elevation in leads I, aVL, and V2 through V6. What coronary artery is the patient having an MI in? C EKG (ii) is incorrect. A Wenckebach has a PR > > 0.20 sec that gradually gets longer then it drops a beat. D EKG (i), (ii), and (iii) are incorrect. See above for explanation. - You've just begun your clinical rotation in the Emergency Department when one of the ER technicians hands you an EKG strip. Glancing over the EKG you noticed that the PR interval is greater than 0.20 sec (EKG i). As you are about to document your findings, two more EKGs are given to you. The first new EKG strip appears to have a constant P-P and R-R interval with a variable P-R interval (EKG ii) and the other EKG strip looks similar to the first EKG, except there is a missing wave associated with ventricular contraction (EKG iii). What conditions are associated with each EKG, (i), (ii), (iii), respectively? A) 1st degree AV block, 3rd degree AV Block, 2nd degree AV block (Mobitz 2) B) 1st degree AV block, 2nd degree AV block (Mobitz 2), 3rd degree AV Block C) 1st degree AV block, 2nd degree AV block (Wenckebach, Mobitz 1), 2nd degree AV block (Mobitz 2) D) 2nd degree AV block (Mobitz 2), 1st degree AV block, 1st degree AV block Correct answer: C. RSR in leads I, V6 would suggest left bundle branch block. RBBB may also have an RSR wave but this would more likely be in leads V1 and V2. LBBB are associated with both acute coronary syndrome and chronic ischemic heart disease, but the description of symptoms is more consistent with ischemic heart disease than an acute coronary event. Those symptoms would more likely be chest pain and tightness that is constant even at rest and that lasts more than 30 minutes and may not have precipitating factors. RBBB can be associated with acute coronary disease, but those are not the symptoms being described and it is not consistent with the EKG findings. - A patient with known history of coronary artery disease presents to the emergency department with complaints of tightness and pressure in the chest that worsens with exercise but is absent at rest. Patient states that that pain will also occur after meals or going outside in the cold, but it doesn't last long. Patient also complains that the pain radiates to the shoulder and back. While reading the patient's EKG you notice RSR waves in leads I and V6. These findings are most consistent with which of the following? A. Right Bundle branch block associated with acute coronary syndrome B. Right bundle branch block associated with ischemic heart disease C. Left bundle branch block associated with ischemic heart disease D. Left bundle branch block associated with acute coronary syndrome A. Though atrial fibrillation can be brought on by electrolyte imbalance, increased age, and alcohol abuse, the ECG as shown is more suggestive of Torsades. *B. With the patient's history of alcohol abuse, increased age, and hypomagnesemia/hypokalemia, combined with the classic look of the ECG showing V tach that twists around baseline, the most likely diagnosis is Torsades.* C. STEMI does not present with classic signs of V tach that twist around baseline. The ECG is more suggestive of Torsades. D. Wolff-Parkinson-White ECGs present with delta waves, wide QRS, and short PR interval. This ECG is more suggestive of Torsades. E. Sinus Bradycardia ECGs present as normal sinus rhythm with a rate less than 60. This ECG is more suggestive of Torsades. - John Limbaugh is a 68 year old alcoholic Caucasian male who presents to the ED via ambulance after a night out drinking heavily with friends. After his twelfth drink of the evening, John complained to his friends of palpitations and feeling faint. He then passed out and EMS was called. On his arrival at the ED, EMS hands you their en-route ECG and you immediately recognize V tach that twists around baseline. Labs drawn show the patient to have hypomagnesemia and hypokalemia. What is your the most likely diagnosis? A. Atrial Fibrillation B. Torsades De Pointes C. STEMI D. Wolff-Parkinson-White E. Sinus Bradycardia Answer: B) A is wrong because that is more indicative of a tension pneumothorax B is correct C is wrong because you can have a pericardial tamponade without a hemo or pneumothorax D is wrong because it is in reference to Tetralogy of Fallot - You are currently rotating in the emergency department when you receive a patient that was involved in a bad motor vehicle accident. He is unconscious and his vital signs are as follows: BP 155/120, RR 18, HR 115, Temp 98.7. Pt has a large contusion over his anterior chest that looks like the impression of a steering wheel. The patient has JVD which extents to the veins in his face. You are highly suspicious of pericardial tamponade. What findings do you expect to see on imaging supporting your differential? A. Midline deviation of the trachea B. Pericardial effusion with collapsed right ventricle C. Hemothorax or pneumothorax D. Bootleg heart shape A. A-fib (this is not it because you cant cause this with a vagal maneuver.) B. 2nd degree block ( the scenerio didnt mention missing p waves) *C. Wandering pacemaker* D. Complete block ( they wouldn't be able to talk about it without calling 911) E. PVC's (this is a change with ventricular waves) - A group of PA students were curious what would happen if they did a vagal maneuver while attached to an EKG machine. They were really worried because what they ended up seeing was a sudden slowing in the SA node followed by an increase back to normal rhythm. Mean while the p wave changed morphology after each beat before returning to normal, this is known as Multifocal rhythm. Another name for this is A. A-fib B. 2nd degree block C. Wandering pacemaker D. Complete block E. PVC's The answer is C because The murmur is caused by turbulent flow through the incompetent mitral valve leaflets into the left atrium. Mitral sounds are best heard at the apex due to the anteriorly directed jets of blood flow striking the lateral wall of the left atrium. Wrong answers: A, creates muffled or distant heart sounds. B, causes absent lung sounds. D, heard over the left sternal border with rumbling character and tricuspid opening snap with wide-splitting S1. Not blowing or systolic. E, High pitched, decrescendo murmur at LUSB, increases with inspiration - Ashley one of the classes more worldly students grew up traveling around the world, most extensively around India and Pakistan. She is in class practicing heart sounds when her class mate hears high-pitched "blowing" holosystolic murmur hear at the tricuspid area and at the mitral area. Ashley has notices some dyspnea on exertion but other than that has had no preceding symptoms. What could be the cause of her murmur? A. Pericardial tampinod B. Pleural effusion C. Mitral regurgitation D. Tricuspid stenosis E. Pulmonic regurgitation The correct answer is B. LV Heart failure. Some of the most common s/s are Dyspnea with exertion ( this is an earlier sign) Orthopnea is a later sign. Pulmonary congestion with rales and rhonchi on lung examination and hypertension. A. With Right Ventricular Heart failure, you would see JVD, Peripheral Edema with GI disturbances possibly including N/V C. Pulmonary Edema is a symptom of the Left Ventricular Heart failure but is not the root cause D. Angina can be a symptom of left ventricular Heart failure but it is not the cause - A 65 year old man presents to clinic with complaint of trouble breathing with exertion and has had a chronic unproductive cough for the past few months. On presentation he is breathing very rapid and shallow and you notice he is sweating profusely. He is taking a Thiazide diuretic for Hypertension but his blood pressure is still elevated. You hear rales and rhonchi on lung sounds and notice a laterally displaced PMI on physical examination. What is most likely the root cause of all your patients symptoms? a. Right Ventricular Heart Failure b. Left Ventricular Heart Failure c. Pulmonary edema d. Angina A is incorrect because it describes a restrictive cardiomyopathy. Additionally, the ejection fraction is preserved in restrictive cardiomyopathy B is incorrect because it describes HOCM, which does not correlate well with the patient's progressive symptoms, her history, or with her age C and D are incorrect because dilated cardiomyopathy with ventricular dilation generally has a decreased ejection fraction. E is the correct answer. - A 62 year old female with a 30-year history of alcohol abuse presents to your primary practice office with complaints of worsening fatigue and shortness of breath. She tells you that for the past six nights, she has been waking suddenly to severe shortness of breath and coughing fits. On exam, you appreciate an S3 heart sound. You are concerned the patient has dilated cardiomyopathy, and order an echocardiogram. Which of the following would you expect the echo to show? A) poor ventricular relaxation with decreased ejection fraction B) septal hypertrophy with decreased ejection fraction C) ventricular dilation with increased ejection fraction D) ventricular dilation with normal ejection fraction E) ventricular dilation with decreased ejection fraction The correct answer is D, pericardial friction rub. The patient is experiencing signs and symptoms of pericarditis. More specifically, Dressler's syndrome, an autoimmune pericarditis that occurs several weeks post MI. Diagnostic testing would likely show diffuse ST elevations and PR depressions on EKG, and pericardial friction rub on physical exam. A) Pericarditis is associated with diffuse ST elevations, not depressions A 62 year old male presents to the emergency room with complaints of dizziness, being light headed, and a syncopal episode earlier today. He has a medical history of hypertension and hyperlipidemia, both of which he is non-compliant with his medications and diet. He does not currently complain of chest pain though he describes an odd fluttering feeling. His vitals are as follows, T: 98.2 F; BP: 91/62; HR: 38 (weak and irregular); R: 16; SPO2: 94% RA. Upon physical exam you note bilateral, weak and irregular pulses. A 12 lead EKG was performed immediately and appeared as attached. Given this patient's presentation and EKG findings, which intrinsic pacemaker of the heart is responsible for this patient's current heart rate? A) Atrial foci B) SA node C) Junctional foci D) Ventricular foci E) None of the above Answer: D) The answer is D because PAC is an arrhythmia because it gives off a premature beat and because of the premature beat, it then is an irregular rhythm. The heart rate can vary according to the underlying rhythm. For PAC there is always an upright P wave in front of the QRS complex because the conduction is still coming from the atria and traveling through the AV node and down the normal path. It's just not the SA node specifically. If a P wave is not seen, then it is hidden in the T wave. B is not correct because an inverted P wave indicates a ventricular electrical origin with the electrical signal traveling in reverse through the AV node. This may happen if the SA node and the AV node have failed to fire, however, for PAC, the signal is still originating somewhere in the RT or LT atria. - You get a phone call from your husband telling you that he's going to the ER because he's experiencing palpitations. You immediately leave work and meet him there. After a short while the ECG is hooked up and after a quick glance at lead II you say, "Aaaaaahhhhh yes". Thinking he's dying, your husband asks what you see. You, being the awesome PA that you are, tell him he's having premature atrial contractions. He wants to know more. You then tell him how you know it's PAC. What are the ECG findings for PAC? (select all that apply) A. Irregular rhythm with premature P waves B. Inverted P waves C. P waves that can be hidden in the T waves D. A and C E. A and B Answer B) People <50 years old diagnostic BNP is >450 pg/mL For people in the age range from 50-75 years old, > 900 pg/mL For people >75 years of age diagnostic BNP is >1800 pg/dL - You are on your elective cardiology rotation in the beautiful state of Minnesota. One of your patients is Norm. Norm is a 55 year old male who loves his Vikings and ice fishing. Norm has an 18 year history of diabetes, is obese and has smoked 1 pack of cigarettes/day since he was 18. He endorses drinking 3 Busch beers per day, and works for the forestry service. CC is wt gain, puffy ankles, shortness of breath (even at night), and occasional chest pain, and constant cough. You suspect he has new onset heart failure and order a BNP, what value would help support your diagnosis for HF? a) 450 pg/mL b) >900 pg/mL c) >750 pg/mL d) BNP is not a good lab to help diagnose HF Answer: A) A is correct because it shows classic physical exam signs and has a history of possible dental infection. B is incorrect. It would cause problems with the valves 10-20 years down the road. C is incorrect because it would not cause a fever. D is incorrect. The physical exam and hx are not consistent with vasculitis. E is incorrect because the pt does not have any peripheral edema. - A 23 yo female presents to your clinic with a complaint of fever, headache and joint pain for two weeks. She states the symptoms started a couple of days after she had a tooth pulled in Mexico (because hey, it's cheaper). Physical exam shows petechiae and janway lesions with no peripheral edema. What condition do you suspect? A. Infective Endocarditis B. Rheumatic Heart Disease C. Deep Vein Thrombosis D. Vasculitis E. Lymphedema Answer: A) Reasoning: A. The most common cause of acute aortic valve regurgitation is infective endocarditis, which typically presents with symptoms such as fever, malaise, cough, pleuritic pain, and dyspnea as stated in the vignette with a common risk factor being over 60 years of age. (Slide 26) B. The most common causes of chronic valve regurgitation are congenital, calcific aortic valve disease, rheumatic heart disease, aortic root dilation (Marfan's, HTN, Familial aortic aneurysm diseases). (Slide 26) C. Tricuspid stenosis is most commonly caused by rheumatic fever, fibrosis, carcinoid syndrome, endocarditis, endomyocardial fibrosis, systemic lupus erythematosus, and congenital tricuspid atresia. This is a mid-diastolic murmur that is low frequency and increases during inspiration. (Slide 74) D. Pulmonic regurgitation is a similar diastolic high pitched murmur that is also heard best at the left upper sternal b - A previously healthy 61-year-old female presents to your clinic complaining of fever, malaise, cough, pleuritic pain and dyspnea. Upon auscultation, you hear crackles in the lungs and in the left upper sternal border, you note a high-pitched blowing murmur during diastole, especially on expiration when leaning forward. You suspect your patient has infective endocarditis, which is the most common cause of: A. Acute aortic valve regurgitation B. Chronic aortic valve regurgitation C. Tricuspid stenosis D. Pulmonic regurgitation E. Aortic stenosis Answer: C) Any maneuver which makes the LV smaller (valsava, standing) results in earlier click and longer murmur duration. Squatting decreases and delays click onset. - A 26-year-old female presents to your clinic who complains of an unusual heart racing sensation when she exercises. She reports chest pain, worse with exertion and caffeine. She states she has been under excess stress lately. Her physical exam is significant for a mid-systolic click. Echocardiogram confirms mitral valve prolapse. When the patient stands, you would expect the murmur to _______. When the patient squats, you would expect the murmur to _______. A: Increase, increase B: decrease, decrease C: increase, decrease D: decrease, increase E: stay the same Answer: A) *A. Sinus Tachycardia - TRUE. This is correct because everything looks and sounds normal with the exception that the rate is above 100 bpm. This is consistent with the fact that she is emotionally stress and has anxiety. B. Ventricular Tachycardia - False. The biggest give away in VT is a prolonged QT interval/very wide QRS complex. It would also show at least 3 consecutive ventricular beats that are faster than normal. This is commonly associated with acute MI's and due to the pt's age, gender, and medical hx, it makes it highly unlikely and the normal EKG proves that. C. Paroxysmal Supraventricular tachycardia - False. Although rate is above 100 bpm and rhythm are normal, in this condition, it would show narrow QRS complexes and P waves difficult to discern. This can also be more of a sudden onset and termination rather than "heart racing during anxiety". So, this option is incorrect for several rea - A 25-year-old female patient comes in your clinic with the complaint of severe anxiety and emotional stress. She states that she feels her "heart is racing"! While listening to her heart, you notice no changes in rate or rhythm with inspiration or expiration. You do an EKG and find all waves normal, regular rhythm, and a rate of 125 bpm. What do you suspect from your findings? A) Sinus Tachycardia B) Ventricular Tachycardia C) Paroxysmal Supraventricular tachycardia D) Sinus arrhythmia Answer: A A- is the correct location (AV junction) and correct presentation of the p-wave- usually absent, and if present inverted. B/D wrong location not the SA node C- correct location but wrong p wave presentation - Daddy Pig has been drinking large amounts of caffeine to keep up with Peppa Pig. He started to feel some palpitations. In the ED you run an ECG and note that he has a sinus rhythm with premature junctional complexes (PJC's). Where does this premature beat originate from? And what's its characteristic "p wave" presentation? A- AV Junction, Absent/ inverted B- SA node, absent/inverted C- AV Junction, present/upright D- SA Node- present/upright Answer: A) Permanent pacemakers are the therapy of choice in patients with symptomatic bradyarrhythmias in sick sinus syndrome. Medications used to treat high blood pressure or heart disease, such as beta-blockers or calcium channel blockers, can worsen abnormal heart rhythms. In some cases, adjusting these medications can relieve symptoms. Radiofrequency ablation is used for the treatment of accessory pathways in the heart. - abdominal/pelvic ultrasound on both patients. Just as you suspected. Both patients have identical diagnoses. What might that diagnosis be? A. Appendicitis B. Acute Gastritis C. Cholelithiasis with Cholecystitis D. Abdominal Aortic Aneurysm E. Diverticulitis Medical reasoning: The correct answer is B. The vein described is a varicose vein which can be removed by varicose stripping. Reticular veins are also caused by weak or damaged valves that allow blood to pool in superficial veins, but they are smaller and do not bulge from the skin as varicose veins do. They are treated by sclerotherapy. The information about telangiectasias is also correct, but it is not what is being described in the question. Telangiectasias are tiny and threadlike. - During a physical examination on a 55-year-old woman, you notice an enlarged, dilated, and tortuous subcutaneous vein that is blue in color and bulging from the patient's left calf. When the patient sits down, the vein is no longer visible. The patient complains that it causes her pain and she hates the way it looks. She asks what it is and if there is any treatment for it. You reply: A. It is a reticular vein, caused by weak or damaged valves that allow blood to pool in superficial veins. They can be removed with sclerotherapy. B. It is a varicose vein, it is caused by weak or damaged valves that allow blood to pool in superficial veins of the leg. It can be removed by varicose vein stripping. C. It is a varicose vein, it is caused by weak or damaged valves that allow blood to pool in superficial veins of the leg, but nothing can be done to remove it. D. It is a telangiectasia, caused by broken blood vessels, and it can be removed with laser therapy. C) Lower back pain results from expansion of the aneurysm. The expansion applies pressure in the abdomen, and the pain is referred to the lower back. Abdominal pain is the most common symptom resulting from impaired circulation. Absent pedal pulses are a sign of no circulation and would occur after a ruptured aneurysm or in peripheral vascular disease. Chest pain usually is associated with coronary artery or pulmonary disease. - A 60- year old male is brought to the ED complaining of a sudden, intense and persistent chest pain and a pulsating feeling in the stomach. Which of the following signs and symptoms usually signifies rapid expansion and impending rupture of an abdominal aortic aneurysm? A. Chest pain B. Absent pedal pulses C. Lower back pain D. Abdominal pain E. None of the above 150-250 = Paroxysmal Tachycardia 60-100 = Normal Sinus Rhythm 60-40 = Junctional automaticity foci 250-350 = Flutter - You are in your ER rotation and your preceptor asks you to analyze an EKG strip from a patient who was admitted. You are able to calculate the rate with a quick glance and tell the preceptor that you are looking a normal sinus rhythm. He is pleased and asks you what is the rate range of a normal sinus rhythm, which you reply with A. 150-250 B. 60-100 C. 60-40 D. 250-350 The answer is all of the above because smoking and hyperlipidemia increase the risks of stroke and embolization. Also, carotid artery angioplasty and stenting as well as endarterectomy are options to consider for treatment due to the symptoms and presentation of the patient already having a TIA history. - Mr. Occlusive, a 67 year old patient presents today to the Emergency Department with recent TIA. He has a history of smoking and hyperlipidemia. He continues to smoke and is noncompliant with treatment of his hyperlipidemia. On auscultation, you hear a carotid bruit and you decide to order a carotid artery duplex. You suspect the patient may have carotid occlusive disease and has a medical history that increases the risk of stroke and embolization. What treatment options might you consider? A) Discuss smoking cessation options B) Discuss other treatment options for hyperlipidemia that may work for the patient C) Carotid Artery Angioplasty and Stenting D) Endarterectomy E) All of the above According to Dr. Plato mesenteric Ischemia lecture, Catheter angiography is the gold stand for detecting this ischemic event. Although it is possible for 53yr old female to be pregnant, this patient has weight loss which would make this choice inconsistent. Although it is possible to experience weight loss with stomach bugs but I didn't include any GI disturbances to warrant a stool sample. Drug test is always useful, now a days you can't assume everyone is drug free. Abdominal wink would not be correct because we don't need to do a neuro reflex exam for this type of pain. - 53yr old female with a 50 pack year history presents to your clinic with stomach pain. She says she is tired of dealing with stomach pain every single time she eats because it lasts 1-3 hrs. Over the last few months, She states she's lost 25lbs and is so scared to eat that she tries to skip meals to avoid pain. What would be the best diagnostic test to determine the source of her pain. A. Abdominal wink B. Catheter Angiography C. Stool sample D. Drug test E. Pregnancy test Correct answer: B The patient meets criteria for stage 1 hypertension based on the two separate clinic blood pressure measurements >130/80. According to the JNC-8 guidelines, the treatment plan should include both healthy lifestyle recommendations and initiation of 1 BP lowering medication based on both the blood pressure readings and the presence of diabetes, which is an increased risk factor for heart disease and stroke. Lifestyle changes without medication intervention are only recommended in cases when the blood pressure is 120-129/80-89. It is not recommend to start the patient on 2 BP lowering medications until the patient reaches stage 2 hypertension, which is diagnosed with blood pressures >140/>90. The JNC-8 guidelines do not advise delaying treatment in patients with increased risk factors and elevated blood pressure. - A 46 year old non-African American male presents to his primary care appointment with a blood pressure reading of 136/85. His medical history includes type 2 diabetes (HgbA1c 8.2) diagnosed 2 years ago and obesity (BMI 34). At his appointment 3 months ago, blood pressure was measured at 133/82. According the JNC-8 guidelines, what should your treatment plan include? A. Recommend healthy lifestyle changes and reassess in 6 months B. Recommend healthy lifestyle changes and begin the patient on 1 BP lowering medication, reassess in 1 month C. Recommend healthy lifestyle changes and begin the patient on 2 BP lowering medications, reassess in 1 month D. Begin the patient on 2 BP lowering medications and reassess in 1 month E. No intervention is necessary because the patient's blood pressure is trending towards normal Reasoning: the patient has many of the hallmark signs of DVT as well as several major risk factors including: period of immobility, recent surgery, pitting edema, Homan's sign. The diagnosis should be easy to get so the next step is choosing treatment. The patient should not be kept still and should even be helped in ambulate in most cases. Antibiotic would not help because this is not an infection. Decompression would not help because this is not a compartment syndrome. Anticoagulation therapy would be a reasonable next step. The surgery may be a relative contraindication but in the absence of other bleeding risks it would not prohibit thrombolytic therapy - You are doing a rotation in oncology and you see a 46 year old patient who has been bed bound in the hospital for 2 days after an spinal disk surgery. The patient is now complaining of pain and tenderness to their right leg. On examination you find the patient's leg is red and swollen with pitting edema, with prominent veins and pain on passive dorsiflexion of the foot. After confirming the most likely diagnosis, what would be a reasonable next treatment? A. Keep the patient in bed and as still as possible to prevent the clot from breaking free. B. Start IV antibiotics to stop the infection C. Anticoagulation therapy D. Surgical decompression of the compartment E. The recent surgery is an absolute contraindication for anticoagulants, only pressure stockings should be used. Answer: A) There are multiple ways to determine axis but one of the easier ways is using the twolead method: The leads you will focus on are leads I and AVF. Focusing in on the QRS complex of these leads you will determine if the QRS complex is positive or negative. If the lead I is positive and Lead aVF is negative this means that there is LAD. However, if lead I is negative but lead aVF is positive there is RAD (Kashou 2018). Based on the image provided above lead I is positive and lead aVF is negative. Therefore, there would be LAD based on these findings. In this particular case the LAD is due to a mechanical shift of the patient's heart since she is currently pregnant. There is no evidence of RAD because lead aVF is not positive and the EKG is not normal because of the evidence of axis deviation. - A 26 yo female comes in for routine labs and an EKG. She is currently 5 months pregnant. Your medical assistant comes back with the EKG and is concerned because there appears to be something wrong with the EKG. The patient is not currently experiencing any symptoms and has no personal or family history of cardiac disease. Based on the EKG below what is the diagnosis and what could be the cause? a) LAD, pregnancy causing a shift in the physiological position of the heart. b) RAD, normal variation c) The EKG is normal You are working in a ER and a 43 year old heavy set man arrives with a history of hypertension. His social history includes use of anabolic steroids queuing you in to the possibility of Left ventricular hypertrophy. What is the procedure of choice to identify Left Ventricular Hypertrophy? a. X-ray b. CT c. Echocardiography d. MRI Answer: B A. The patient is suffering from primary hypertension due to old age. (Although older patients can get increase of their blood pressure, an increase from normal to 180/110 is highly abnormal without any known cause) *B. The patient is suffering from secondary hypertension due to his recent weight change causing his sleep apnea. (Slide 46 specifies likely causes in this age group one being sleep apnea) C. The patient is suffering from secondary hypertension due to renal parenchyma disease. (Slide 46 specifies this is the main cause of secondary HTN for kids under 18) D. The patient is suffering from primary hypertension because his wife is constantly nagging him about his weight. E. The patient is suffering from secondary hypertension and it is irreversible. (Slide 43 states secondary hypertension is from a reversible cause) - A 64 year old male presents to the clinic with his wife for his annual checkup. He has had no significant previous medical history. Upon reviewing his social history, you learn that the patient won the lottery and has been going on multiple cruises. His wife yells out that he has gained at least 15 lbs already. She also complains that he has been snoring very loudly. His vitals are: Ht: 5'6, Wt: 220, B/P 180/110, RR20, BPM 90, O2 98%, Temp: 98.7F. This is surprising to the patient as he has no personal history or family history of high blood pressure. What is likely going on with the patient? A. The patient is suffering from primary hypertension due to old age. B. The patient is suffering from secondary hypertension due to his recent weight change causing his sleep apnea. C. The patient is suffering from secondary hypertension due to renal parenchyma disease. D. The patient is suffering from primary hypertension because his wife is constantly nagging him about his weight. E. The patient is suffering from secondary hypertension and it is irreversible. (Slide 43 states secondary hypertension is from a reversible cause) Answer: B) a. Initial ventricular depolarization i. This answer is incorrect because ventricular depolarization is represented in the QRS complex, before the ST segment where they are contracting. b. *Initial ventricular repolarization i. This answer is correct as the ventricles depolarize in the QRS complex and they begin to repolarize in this segment. The final ventricular repolarization period is represented in the T wave. c. Atrial depolarization i. This answer is incorrect as atrial depolarization represents atrial contraction and is represented by the P wave, prior to the QRS complex. d. It is usually not present in an EKG i. This answer is false as it should be present in an EKG and is an important factor in identifying pathology and imminent problems. - A patient comes to the emergency room complaining of sharp, sudden chest pain that started 30 minutes ago. The tech hands you the EKG and you begin to read it. The doctor you are shadowing asks you to identify the ST segment and what it represents. You respond: a. Initial ventricular depolarization b. Initial ventricular repolarization c. Atrial depolarization d. It is usually not present in an EKG The answer is A. All dissections result from a spontaneous tear of the tunica intima from the tunic media. This tearing of the layers of the aorta creates a channel that can continue to rip away creating a true and a false lumen. This is different from an aortic aneurysm; the intimal wall remains intact, and each layer of the aorta is weakened to the point where it stretches beyond it's normal diameter. Also, risk factors include HTN, age, and deceleration injuries! - Your first night in the ER as a PA-S you receive a 50 y/o male with a Hx of HTN whose airbag did not deploy in a head-on collision (each going ~30mph). The Pt c/o severe chest pain that radiates to his jaw, neck and upper back pain that he says is "tearing." This may signify that the tunica ___________ separated from the tunica ___________ resulting in an aortic _____________. He just went syncopal. A. Intima.... Media.... Dissection B. Media.... Adventitia.... Dissection. C. Media.... Adventitia.... Aneurysm. D. Intima.... Media... Aneurysm C) I formed the vignette based on common symptoms of DVT listed in slide 12. On the following slide, standard therapies are listed. A is correct as it is a form of parenteral heparin and should be continued for 5 days, Coumadin should be started as well so B is also correct, Apixaban is an acceptable NOAC alternative to Coumadin, making E correct and D is correct since compression stockings should be worn for at least 2 years. C is the correct answer, early ambulation is important in therapy for DVTs. I found an meta-analysis article (below) that helps support the benefits of early ambulation. Bed rest has been practiced in that past to prevent PE, but the study found no association of early ambulation in acute DVT patients to higher incidence of new PE. Early ambulation can also have improved outcomes on pain in DVT patients. - During your clinical rotations, you encounter a 63 year-old patient with complaints of pain, edema, erythema, and tenderness of her right calf. You perform a duplex ultrasound and make your first independent diagnosis! Your preceptor is impressed and is confident you will decide the best course of treatment for this patient. Which of the following is NOT a standard therapy for this diagnosis? A. Low molecular-weight heparin for 5 days B. Coumadin C. Bed rest with right leg elevation D. Compression stockings E. Apixaban A: Thickened skin is not a symptom of peripheral artery disease B. Bounding pulse in affected extremity is not a symptom of peripheral artery disease C. With an ABI of 0.6, a common sign of peripheral artery disease would be pallor of the affected extremity when elevated, as blood flow to the extremity would be decreased D. Dusky red coloring of affected extremity is not a symptom of peripheral artery disease - A 55 y/o male comes into your clinic with complaints of pain in his lower left leg when walking. He states the pain improves when he stops walking. He finds that the pain goes away when hanging his foot off the edge of the bed. When asked to walk around in the clinic, the patient reports returning of pain symptoms in his left lower leg. On physical exam, the patient's ankle-brachial index (ABI) is 0.6. What other physical findings would be evident given the patient's mostly likely diagnosis? A. Thickened lower left skin B. Bounding pulse in left lower leg C. Pale coloring when left lower extremity is elevated D. Dusky red color when left lower extremity is elevated B is the correct answer because secondary lymphedema is associated with cancer surgery, starts distally, and is rarely painful. Pitting is associated with the early stages of lymphedema. A is incorrect because there need to be signs of inflammation that indicates an infection. C is incorrect because there is no ulceration which is more commonly found in venous diseases. D is incorrect because it is associated with recent surgery, is painful. - 59 yo Jane present to your primary care facility with complaints of L leg swelling and discomfort without pain. On physical exam, you noticed squared toes, pitting in her L foot, and mild swelling in her leg. Past medical history includes a total hysterectomy 3 years ago. The swelling in the leg is most likely due to A) lymphangitis B) lymphedema C) varicose veins D) deep vein thrombosis B - 35 year old man presents with a heart racing sensation. He reports he has been drinking heavily with friends over the weekend and now complains of chest pain. You perform a 12 lead EKG on the patient. When determining the rate you count 2 large boxes between the R-R, what is the patient's heart rate? A: 300 B: 150 C: 100 D: 75 E: 60 Reasoning: All of the other options are correct in support of a popliteal aneurysm. - 65 year old male patient has a history of smoking, DM, and HTN. Patient is now complaining of pain to his right leg upon exertion but feels better upon rest. Physical Examination shows that there are no palpable pulses and loss of hair on that limb. Your differential diagnosis includes popliteal aneurysm; what other symptoms would expect to find? labs are within normal limits. You established that he is having a hypertensive urgency. What is your treatment plan for this patient after he is discharged? A. Immediately reduce the patient's BP and admit to ICU B. Start on 2-drug oral combination that will be continued outpatient C. Start the patient on Sodium Nitroprusside D. Do nothing The correct answer is C. The P wave represents atrial depolarization and the QRS complex represents ventricular depolarization. This electrical activity is directly related to the mechanical function of the heart. The other options are incorrect because although it is true that they are also part of a normal EKG / cardiac cycle, they do not accurately define what a P wave and a QRS complex represent. - A 50y/o F arrives at your clinic complaining of an abrupt onset of dizziness that began approximately one hour ago. As part of a full work-up you order a 12-lead EKG for this patient. While informing her that the EKG looked normal, she asks you "what do all those lines mean anyways." You confidently explain, this first small bump is a P wave and represents ___________ ____________, and this big spike is called a QRS complex which represents _______________ ______________, both of which make the heart contract. A. Atrial Repolarization; Atrial Depolarization B. Ventricular Depolarization; Atrial Depolarization C. Atrial Depolarization; Ventricular Depolarization D. Atrial Repolarization; Ventricular Repolarization C) A PR interval < 0.12 can be indicative of an impulse that originated somewhere other than the SA node (junctional arrhythmias, and preexcitation syndromes) A PR interval > 0.20 seconds may represent a conduction delay through the atria or AV junction resulting from digoxin toxicity or heat block Option A is wrong, that is the correct range for a normal P wave Option B is incorrect Option D is incorrect, that is the correct range for a QRS complex Option E is incorrect because the appropriate answer does exist (option C) - An eighty-year-old male patient comes into your ED complaining of chest pain. The great triage tech immediately takes the patient back for an EKG. The tech hands the EKG to the attending and hears the attending muttering about "prolonged PR intervals". Being interested in cardiology, the triage techs goes to Dr. Google and researches a bit on prolonged PR intervals. She discovers that a prolonged PR interval can be a result form digoxin toxicity, among other things. What is the normal range for a PR interval? A. 0.06- 0.12 seconds B. 0.10 - 0.24 seconds C. 0.12 - 0.20 seconds D. 0.06 - 0.10 seconds E. None of the above Correct answer: C A left axis deviation would have a positive QRS in lead 1 and negative in AVF. A normal range would be positive in both There is enough information to answer the question - What is the electrical axis for an EKG whose QRS is mainly positive in AVF and mainly negative in lead 1? A. Left axis deviation B. Normal Range C. Right axis deviation D. Not enough information A) The correct answer. In A Fib you do not see a P wave B) NSTEMI EKG's would have a noticeable ST depression. C) She would still have a P wave if she was having an anxiety attack. D) Wandering pacemaker you see P waves, but they are irregular in their shape. - Jill Jackson is a 67 year old newly retired schoolbus driver. Jill has had hypertension for 7 years now, and is compliant with her low sodium diet. However, she does not take her Losartan as prescribed by her PA-C. Today, she comes into the ED with new onset chest pain and a near syncopal episode. Upon admission, she was hooked up to a 12 lead EKG. Her EKG showed a very irregular ventricular rhythm (QRS complex) and a HR of 70 BPM. You notice that there are no P waves on the EKG strip. You immediately suspect this patient is suffering from: A) Atrial Fibrillation B) NSTEMI C) Anxiety attack D) Wandering Pacemaker. C/D are wrong since this is usually done when ultrasound is available B) Correct Dx test but 60% is considered moderate- antiplatelet therapy is indicated A) Correct Dx and severe occlusion is indicated for surgery. - Bob The builder presents to the clinic with visual disturbances that occured a day ago, he described it as a dark shadow to his left eye, but normal vision returned after a few minutes. Prior Hx includes smoking, diabetus, and a left sided bruit. What is the initial diagnostic test indicated (all equipment available) and at what percent is surgery indicated. A. Duplex Doppler Ultrasound- 75% occlusion B. Duplex Doppler Ultrasound- 60% occlusion C. Angiography- 60% occlusion D. Angiography- 75% occlusion The Correct answer is B; Primary hypertension is due to an idiopathic etiology with onset around 25-55 yrs. Sometimes due to Fhx Secondary Hypertension is due to an underlying cause and can often be corrected. Secondary should be considered if blood pressure is refractory to antihypertensives or severely elevated. Correct Answer There is not enough information to diagnose CVD CHF could include symptoms of SOB, fatigue, Edema in legs and elevated HR - A 30 year old male walks into clinic for a general visit and checkup. The MA records his height as 5'11" and weight at 165, his blood pressure at 145/86. The patient stated the last time he was in clinic that the values were about the same, the MA confirmed that they were. When you walk in and look at the results you find that there is no underlying cause of his readings. What would be the most likely diagnosis? A. Secondary Hypertension B. Primary(Essential) Hypertension C. Cardiovascular disease D. CHF The answer is B 150 beats per minute. The hardest part of this question is just understanding the pattern with the rule of 300. Answer A is incorrect because there would only be one big box between the peaks. C is incorrect because it would be about two and a half box's. D is incorrect because it would be in between one and two box's. E is incorrect because you most definitely can calculate the rate given the prior information. - You are currently rotating in the Emergency Department when a patient is brought in by ambulance due to a symptomatic bout of supraventricular tachycardia (SVT). For some unknown reason the pulse oximeter is not working so you can't easily see the patients heart rate on the monitor. You remember from school that you can use an EKG tracing and the rule of 300 to accurately calculate the patients heart rate. The strip shows that there are two large boxes between subsequent QRS complex peaks. You note that the rhythm is... A. 300 BPM B. 150 BPM C. 120 BPM D. 200 BPM E. Unable to calculate given the previous information. A. Q wave First downward wave of the QRS complex, which represents ventricular depolarization B. ST segment Plateau phase of ventricular repolarization C. U wave represents repolarization of the Purkinje fibers D. X wave. Imaginary E. *None of the above. Trick question Atrial repolarization is berried in the QRS complex. - As a highly trained PA student you are aware that an EKG tracing is more than just squiggly lines. If a "P" wave represents Atrial depolarization and a "QRS" represents ventricular depolarization and the "T" wave represents Ventricular repolarization what wave represents Atrial repolarization? A. Q wave B. ST segment C. U wave D. X wave. E. None of the above. This is a young/ healthy patient presenting with numerous syncopal events provoked by physical activity. The vital signs are within normal limits, and the EKG shows a sinus rhythm with QRS voltage increase. This taken along with the systolic ejection murmur is indicative of a left ventricular hypertrophy. The way to diagnose a ventricular hypertrophy is an echocardiogram. Answer C is the correct answer here. An angiogram is not diagnostic for ventricular hypertrophy. - A 28 year old male presents to the emergency department after passing out. He was with friends who witnessed the collapse, and the patient admits to several similar events in the past year with each occurring soon after a physically strenuous activity. Vital signs are all within normal limits and a 12 lead EKG revealed a normal sinus rhythm at a rate of 67 with marked increase in QRS voltage. A moderate systolic ejection murmur is heard at the left sternal border, and is decreased when the patient is in a squatting position. What condition are you most suspicious of, and what diagnostic might be useful to utilize? A) Angina/angiogram B) Aortic stenosis/ echocardiogram C) Left ventricular hypertrophy/ echocardiogram D) Left ventricular hypertrophy/ angiogram E) Mitral regurgitation/ TEE
Docsity logo



Copyright © 2024 Ladybird Srl - Via Leonardo da Vinci 16, 10126, Torino, Italy - VAT 10816460017 - All rights reserved